Những câu hỏi liên quan
thánh yasuo lmht
Xem chi tiết
Thắng Nguyễn
11 tháng 2 2017 lúc 21:45

Câu hỏi của phan tuấn anh - Toán lớp 9 - Học toán với OnlineMath cái này y hệt, tham khảo đi nếu vẫn chưa làm dc thì nhắn cho mk

Bình luận (0)
Uyên Hoàng
Xem chi tiết
Yim Yim
5 tháng 6 2018 lúc 16:28

\(x+\sqrt{x+yz}=x+\sqrt{x\left(x+y+z\right)+yz}=x+\sqrt{x^2+yz+x\left(z+y\right)}\)

\(\ge x+\sqrt{2\sqrt{x^2yz}+x\left(y+z\right)}=x+\sqrt{x\cdot2\sqrt{yz}+x\left(y+z\right)}=x+\sqrt{x\left(y+z+2\sqrt{yz}\right)}\)

\(=x+\sqrt{x\left(\sqrt{y}+\sqrt{z}\right)^2}=x+\sqrt{x}\left(\sqrt{y}+\sqrt{z}\right)\)

\(\Rightarrow\frac{x}{x+\sqrt{x+yz}}\le\frac{x}{x+\sqrt{x}\left(\sqrt{y}+\sqrt{z}\right)}=\frac{\sqrt{x}}{\sqrt{x}+\sqrt{y}+\sqrt{z}}\)

tương tự :

\(\frac{y}{y+\sqrt{y+xz}}\le\frac{\sqrt{y}}{\sqrt{y}+\sqrt{x}+\sqrt{z}}\)

\(\frac{z}{z+\sqrt{z+xy}}\le\frac{\sqrt{z}}{\sqrt{z}+\sqrt{x}+\sqrt{y}}\) 

cộng vế theo vế ta được 

\(\frac{x}{x+\sqrt{x+yz}}+\frac{y}{y+\sqrt{y+zx}}+\frac{z}{z+\sqrt{z+xy}}\le\frac{\sqrt{x}+\sqrt{y}+\sqrt{z}}{\sqrt{x}+\sqrt{y}+\sqrt{z}}=1\)

dấu "=" xảy tra khi x=y=z=1/3

Bình luận (0)
Nguyễn Thị Bảo Anh
28 tháng 3 2020 lúc 21:24

cái này thì chịu

Bình luận (0)
 Khách vãng lai đã xóa
Thiên bình đáng yêu
25 tháng 5 2020 lúc 17:48

khó muốn chết luôn làm sao làm đc

Bình luận (0)
 Khách vãng lai đã xóa
bui thai hoc
Xem chi tiết
tth_new
29 tháng 9 2019 lúc 9:18

Theo em bài này chỉ có min thôi nhé!

Rất tự nhiên để khử căn thức thì ta đặt \(\left(\sqrt{x};\sqrt{y};\sqrt{z}\right)=\left(a;b;c\right)\ge0\)

Khi đó \(M=\frac{a^3}{a^2+ab+b^2}+\frac{b^3}{b^2+bc+c^2}+\frac{c^3}{c^2+ca+a^2}\) với abc = \(\sqrt{xyz}=1\) và a,b,c > 0

Dễ thấy \(\frac{a^3}{a^2+ab+b^2}+\frac{b^3}{b^2+bc+c^2}+\frac{c^3}{c^2+ca+a^2}=\frac{b^3}{a^2+ab+b^2}+\frac{c^3}{b^2+bc+c^2}+\frac{a^3}{c^2+ca+a^2}\)

(chuyển vế qua dùng hằng đẳng thức là xong liền hà)

Do đó \(2M=\frac{a^3+b^3}{a^2+ab+b^2}+\frac{b^3+c^3}{b^2+bc+c^2}+\frac{c^3+a^3}{c^2+ca+a^2}\)

Đến đây thì chứng minh \(\frac{a^3+b^3}{a^2+ab+b^2}\ge\frac{1}{3}\left(a+b\right)\Leftrightarrow\frac{2}{3}\left(a-b\right)^2\left(a+b\right)\ge0\)(đúng)

Áp dụng vào ta thu được: \(2M\ge\frac{2}{3}\left(a+b+c\right)\Rightarrow M\ge\frac{1}{3}\left(a+b+c\right)\ge\sqrt[3]{abc}=1\)

Vậy...

P/s: Ko chắc nha!

Bình luận (0)
bui thai hoc
30 tháng 9 2019 lúc 9:59

dit me may 

Bình luận (1)
Lãnh Hàn Thiên Kinz
19 tháng 7 2020 lúc 19:01

bạn bui thai hoc sao lại cmt linh tinh vậy :)) bạn ko có học thức à :> mà ý bạn cmt như vậy là sao hả ? 

Bình luận (0)
 Khách vãng lai đã xóa
Đặng Phương Nga
Xem chi tiết
Kudo Shinichi
10 tháng 10 2019 lúc 20:51

Theo giả thiết \(\sqrt{\frac{yz}{x}}+\sqrt{\frac{xz}{y}}+\sqrt{\frac{xy}{z}}=3\)

\(\Rightarrow\frac{yz}{x}+\frac{xz}{y}+\frac{xy}{z}+2x+2y+2z=9\)

Mặt khác , ta có BĐT phụ : \(\frac{yz}{x}+\frac{xz}{y}+\frac{xy}{z}\ge x+y+z\)

\(\Rightarrow9\ge3\left(x+y+z\right)\)

\(\Leftrightarrow x+y+z\le3\)

Áp dụng BĐT Cauchy Shwarz \(\Rightarrow\left(\sqrt{x}+\sqrt{y}+\sqrt{z}\right)^2\le3\left(x+y+z\right)\le9\)

\(\Rightarrow\sqrt{x}+\sqrt{y}+\sqrt{z}\le3\)

Ta có : \(P=\sqrt{x}+\sqrt{y}+\sqrt{z}+\frac{2016}{\sqrt{x}+\sqrt{y}+\sqrt{z}}\)

\(=\sqrt{x}+\sqrt{y}+\sqrt{z}+\frac{9}{\sqrt{x}+\sqrt{y}+\sqrt{z}}+\frac{2007}{\sqrt{x}+\sqrt{y}+\sqrt{z}}\)

\(\ge2.\sqrt{9}+\frac{2007}{3}=675\)

Dấu "=" xảy ra \(\Leftrightarrow x=y=z=1\)

Chúc bạn học tốt !!!

Bình luận (0)
liên hoàng
Xem chi tiết
Nguyễn Mai
Xem chi tiết
Phùng Minh Quân
5 tháng 12 2019 lúc 20:40

\(Q=\Sigma\frac{x^4}{x^2+\sqrt{xy.zx}}\ge\frac{\left(x^2+y^2+z^2\right)^2}{x^2+y^2+z^2+xy+yz+zx}\ge\frac{x^2+y^2+z^2}{2}\ge\frac{\left(x+y+z\right)^2}{6}=\frac{3}{2}\)

Dấu "=" xảy ra khi x=y=z=1 

Bình luận (0)
 Khách vãng lai đã xóa
Nguyễn Minh Khuê
Xem chi tiết
Trần Thị Bảo Trân
2 tháng 10 2016 lúc 18:44

Ta có \(x+y+z=1\Rightarrow x+y=1-z,\) ta có:

\(\frac{x+y}{\sqrt{xy+z}}=\frac{1-z}{\sqrt{xy+1-x-y}}=\frac{1-z}{\sqrt{\left(1-x\right)\left(1-y\right)}}\)

\(\frac{y+z}{\sqrt{yz+x}}=\frac{1-x}{\sqrt{yz+1-y-z}}=\frac{1-x}{\sqrt{\left(1-y\right)\left(1-z\right)}}\)

\(\frac{z+x}{\sqrt{zx+y}}=\frac{1-y}{\sqrt{zx+1-x-z}}=\frac{1-y}{\sqrt{\left(1-x\right)\left(1-z\right)}}\)

Khi đó \(P=\frac{x+y}{\sqrt{xy+z}}+\frac{y+z}{\sqrt{yz+x}}+\frac{z+x}{\sqrt{zx+y}}=\frac{1-z}{\sqrt{\left(1-x\right)\left(1-y\right)}}+\frac{1-x}{\sqrt{\left(1-y\right)\left(1-z\right)}}+\frac{1-y}{\sqrt{\left(1-x\right)\left(1-z\right)}}\)

               \(\ge3\sqrt[3]{\frac{1-z}{\left(1-x\right)\left(1-y\right)}\times\frac{1-x}{\left(1-y\right)\left(1-z\right)}\times\frac{1-y}{\left(1-x\right)\left(1-z\right)}}=3\)

Vậy \(MinP=3\) đạt được khi \(x=y=z=\frac{1}{3}\) 

Bình luận (10)
Kuro Kazuya
14 tháng 5 2017 lúc 23:51

\(P=\dfrac{x+y}{\sqrt{xy+z}}+\dfrac{y+z}{\sqrt{yz+x}}+\dfrac{z+x}{\sqrt{xz+y}}\)

\(P=\dfrac{x+y}{\sqrt{xy+\left(x+y+z\right)z}}+\dfrac{y+z}{\sqrt{yz+\left(x+y+z\right)x}}+\dfrac{x+z}{\sqrt{zx+\left(x+y+z\right)y}}\)

\(P=\dfrac{x+y}{\sqrt{xy+xz+yz+z^2}}+\dfrac{y+z}{\sqrt{yz+x^2+xy+xz}}+\dfrac{x+z}{\sqrt{xz+xy+y^2+yz}}\)

\(P=\dfrac{x+y}{\sqrt{\left(x+z\right)\left(y+z\right)}}+\dfrac{y+z}{\sqrt{\left(x+y\right)\left(x+z\right)}}+\dfrac{x+z}{\sqrt{\left(x+y\right)\left(y+z\right)}}\)

Áp dụng bất đẳng thức Cauchy - Schwarz

\(\Rightarrow P\ge3\sqrt[3]{\dfrac{\left(x+y\right)\left(y+z\right)\left(x+z\right)}{\sqrt{\left(x+y\right)^2\left(y+z\right)^2\left(x+z\right)^2}}}=3\sqrt[3]{\dfrac{\left(x+y\right)\left(y+z\right)\left(x+z\right)}{\left(x+y\right)\left(y+z\right)\left(x+z\right)}}=3\)

\(\Rightarrow P\ge3\)

Vậy \(P_{min}=3\)

Dấu " = " xảy ra khi \(x=y=z=\dfrac{1}{3}\)

Bình luận (0)
Thân thi thu
Xem chi tiết
Bùi Hữu Vinh
Xem chi tiết
Yen Nhi
5 tháng 1 2021 lúc 23:17
Bạn tham khảo lời giải của tớ nha!

Bài tập Tất cả

Bình luận (0)
 Khách vãng lai đã xóa